your neighborhood fast food restaurant only sells chicken nuggets in packs of five or seven. what is the largest number of nuggets that is impossible to order?

Answers

Answer 1

The largest number of chicken nuggets that is impossible to order from your neighborhood fast food restaurant is eleven.

The largest number of chicken nuggets that is impossible to order from your neighborhood fast food restaurant is eleven. This is because the restaurant only sells nuggets in packs of five and seven. This means that if you wanted to order eleven nuggets, it would not be possible.

To explain this mathematically, eleven can be written as 5+5+1, 7+4, or 3+3+3+2. As none of these can be made with just five and seven packs of chicken nuggets, it is impossible to order eleven.

If the restaurant sold chicken nuggets in packs of three and seven, then it would be possible to order eleven nuggets (3+3+3+2).

In conclusion, the largest number of chicken nuggets that is impossible to order from your neighborhood fast food restaurant is eleven.

See more about the largest number at: https://brainly.com/question/19267346

#SPJ11


Related Questions

the ages of students at a university are normally distributed with a mean of 20. what percentage of the student body is at least 20 years old?

Answers

The percentage of the student body that is at least 20 years old is 50%.

The ages of students at a university are normally distributed with a mean of 20. To find the percentage of the student body that is at least 20 years old solution to the is as follows: As the Mean of ages of students at a university, μ = 20. Age is normally distributed so it will be divided into two groups  Therefore, the mean age is equal to the median age. As the age is normally distributed, 50% of the population will have an age greater than or equal to 20 years old. Thus, the percentage of the student body that is at least 20 years old is 50%. Hence, the required percentage of the student body that is at least 20 years old is 50%.

To learn more about "Normally distributed": brainly.com/question/14916937

#SPJ11

11. Hannah recorded the number of miles she jogged. She started jogging 3 41 miles. Every week she jogged an additional
1 21 miles. Select all true statements. The y-intercept is the rate of change. The y-intercept is the starting value. The slope is 121. The slope is 3 41. The y-intercept is 1 21. The y-intercept is 3 41

Answers

Using coordinate geometry,

The y-intercept is the starting value (true).The slope is 1/2 or 0.5, not 121 or 3/41 (false).The y-intercept is 3/41 is also false as it contradicts the first true statement, therefore, the y-intercept is the starting value and the y-intercept of 3/41 is also true.

The problem provides two pieces of information about Hannah's jogging routine: she started with 3 41 miles and added 1/21 miles every week. We can use this information to create a linear equation that represents the number of miles Hannah jogged as a function of the number of weeks she has been jogging.

To create this equation, we first identify the starting value, which is 3 41 miles. This is also the y-intercept of the line. Next, we determine the slope of the line, which is the rate at which the number of miles increases each week. The slope is equal to the change in y (miles) divided by the change in x (weeks), which in this case is (1/21 - 3/41)/1 = -2/1 = -2. Therefore, the slope of the line is -2.

Putting these two pieces of information together, we get the equation y = -2x + 3 41, where y is the number of miles jogging and x is the number of weeks of jogging. This is a linear equation in slope-intercept form, where the slope is -2 and the y-intercept is 3 41.

Learn more about the co-ordinate geometry at

https://brainly.com/question/21094061

#SPJ4

I need help with this problem. Joe bought a gallon of gasoline for 2. 85 per gallon and c cans of oil for 3. 15 per can

Answers

From the given information provided, the expression that need to determine the total amount is Total cost = $2.85/gallon x g gallons + $3.15/can x c cans.

The expression that can be used to determine the total amount Joe spent on gasoline and oil is:

Total cost = Cost of gasoline + Cost of oil

We can represent the cost of gasoline as:

Cost of gasoline = price per gallon x number of gallons

Substituting the given values, we get:

Cost of gasoline = $2.85/gallon x g gallons

Similarly, we can represent the cost of oil as:

Cost of oil = price per can x number of cans

Substituting the given values, we get:

Cost of oil = $3.15/can x c cans

Putting it all together, we get:

Total cost = $2.85/gallon x g gallons + $3.15/can x c cans

Expression that can be used to determine the total amount Joe spent on gasoline and oil is:

Total cost = $2.85/gallon x g gallons + $3.15/can x c cans

Question - Joe bought g  gallons of gasoline for $2.85 per gallon and c  cans of oil for  $3.15 per can. What expression can be used to determine the total amount Joe spent on gasoline and oil?

Learn more about amount here: brainly.com/question/25720319

#SPJ4

julio has $31.00 he earns half of that much mowing a lawn. How much money does he have in all?

Answers

Answer: $ 46.50

First divided 31 by 2

Which equals...

15.50

Then add 15.50 to 31.

46.50

The answer is $46.50

On a certain hot summers day, 402 people used the public swimming pool. The daily prices are 1. 50$ for children and 2. 00$ for adults. The receipts for admission totaled 648. 50$. How many children and how many adults swam at the public pool that day?

Answers

From the given information provided, there were 311 children who swam at the pool that day.

Let's use C to represent the number of children who swam at the pool and A to represent the number of adults who swam at the pool.

From the problem, we know that:

C + A = 402 (Equation 1) (The total number of people who used the pool was 402)

1.50C + 2.00A = 648.50 (Equation 2) (The total receipts from admission were $648.50)

To solve for C and A, we need to eliminate one of the variables. We can do this by multiplying Equation 1 by 1.50, which will give us:

1.50C + 1.50A = 603 (Equation 3)

Now we can subtract Equation 3 from Equation 2 to eliminate C:

2.00A - 1.50A = 648.50 - 603

0.50A = 45.50

A = 91

So there were 91 adults who swam at the pool that day. To find the number of children, we can substitute A = 91 into Equation 1:

C + 91 = 402

C = 311

Learn more about variable here: brainly.com/question/82796

#SPJ4

The graphs of line a and b are shown in this coordinate grid
Match each line with it's equation. Drag each equation to the corresponding box for each line

Answers

The correct option for the given graph is option 1 and option 3. The equation matches the intersection point (1,1).

For option 1: intersection point (1,1)

substitute the values of x & y in the given equation.

1 = 3 (1) - 2

1 = 1

LHS = RHS

For option 2: point (1,1)

substitute the values of x & y in the given equation.

1 = 2 (1) + 3

1 = 5

LHS ≠ RHS

For option 3: point (1,1)

substitute the values of x & y in the given equation.

1 = -2 (1) + 3

1 = 1

LHS = RHS

For option 4: point (1,1)

substitute the values of x & y in the given equation.

1 = - [tex]\frac{1}{2}[/tex] (1) + 3

1 = [tex]\frac{5}{2}[/tex]

LHS ≠ RHS

For option 5: point (1,1)

substitute the values of x & y in the given equation.

1 = - [tex]\frac{1}{3}[/tex] (1) + 3

1 = [tex]\frac{8}{9}[/tex]

LHS ≠ RHS

Therefore, correct option for the given graph is option 1 and option 3.

Learn more about Graphs:

https://brainly.com/question/29192564

#SPJ1

2. Claire earns $92, 400 a year gross pay as a company president. She has 5%of her gross pay deposited into a 401(k) retirement plan. How much money does Claire's company deposit into her 401(k)
retirement plan each month?
$300
$385
$275
$325

Answers

Therefore , the solution of the given problem of unitary method comes out to be choice B $385 is the correct response.

An unitary method is what ?

The objective can be accomplished by using what was variable previously clearly discovered, by utilizing this universal convenience, or by incorporating all essential components from previous flexible study that used a specific strategy. If the anticipated claim outcome actually occurs, it will be feasible to get in touch with the entity once more; if it isn't, both crucial systems will undoubtedly miss the statement.

Here,

We must first determine how much is deducted from Claire's gross salary annually for the 401(k) plan in order to determine

how much money is deposited into her retirement account by her employer each month.

Since we are aware that Claire contributes 5% of her total income to her 401(k),

we can figure out how much she contributes as follows:

=>  0.05 x $92,400 = $4,620

As a result, Claire's 401(k) plan deducts $4,620 from her total income each year. We can reduce this amount by 12 (the number of months in a year) to determine how much it is per month:

=> $4,620 ÷ 12 = $385

As a result, Claire's employer contributes $385 each month to her 401(k) savings account.

Therefore, choice (B) $385 is the correct response.

To know more about unitary method  visit:

https://brainly.com/question/28276953

#SPJ1

While birdwatching, Vicki saw 7 red birds, 9 white birds, and 19 black birds. Johnny saw 9 red birds, 12 blue birds, and 29 brown birds. Estimate how many more birds Johnny saw by rounding all numbers to the nearest ten before adding and subtracting.

Answers

Answer:

10

Step-by-step explanation:

7 rounds to 10

9 rounds to 10

19 rounds to 20

so Vicki has 40 birds

9 rounds to 10

12 rounds to 10

29 rounds to 30

so johnny has 50 birds

now subtract 50-40=10

f scores are normally distributed with a mean of 35 and a standard deviation of 10, what percent of the scores is: (a) greater than 34?

Answers

The percentage of scores greater than 34 is 0.5398 or 53.98%.

Given that the mean of scores (μ) = 35 and the standard deviation (σ) = 10. We need to find the percentage of scores greater than 34. Since the scores are normally distributed, we can standardize the variable by using the z-score formula.

z = (x - μ) / σ

Here, x = 34, μ = 35 and σ = 10z = (34 - 35) / 10z = -0.1

We need to find the area to the right of the z-score line on the standard normal distribution table. The standard normal distribution table provides the probabilities corresponding to the z-scores, i.e. the area under the curve to the right or left of the z-score line on the distribution table. The area to the right of the z-score line represents the percentage of scores that are greater than the given value. Using the standard normal distribution table, the area to the right of the z-score line -0.1 is 0.5398.

The percentage of scores greater than 34 is 0.5398 or 53.98%.

To learn more about percentage of scores refer :

https://brainly.com/question/25361715

#SPJ11

a 336-m long fence is to be cut into pieces to make three enclosures, each of which is square. how should the fence be cut up in order to minimize the total area enclosed by the fence?

Answers

The fence ought to be cut into 12 pieces, every one of length 28 m, to make three squares, each with a side length of 28 m. This will limit the total area encased by the fence.

To limit the total area encased by the fence, the three squares ought to have equivalent areas. Let x be the length of each side of the squares. Then the perimeter of each square is 4x, and the total length of the fence is 3(4x) = 12x. Since the total length of the fence is given to be 336 m, we have:

12x = 336

Addressing for x, we get:

x = 28

Find out more about fence

brainly.com/question/14785438

#SPJ4

a bag contains red balls, green balls, and yellow balls. if balls are drawn one at a time without replacement, the probability that the first yellow ball is drawn on the eighth draw is , what is the value of ?

Answers

The probability of drawing the first yellow ball on the eighth draw is 1/2772.

The probability of drawing a yellow ball on the eighth draw is the probability of drawing 7 non-yellow balls followed by a yellow ball.

The probability of drawing a non-yellow ball on the first draw is 7/12 since there are 7 non-yellow balls out of a total of 12 balls in the bag. After the first non-yellow ball is drawn, there will be 6 non-yellow balls left out of a total of 11 balls. So the probability of drawing a non-yellow ball on the second draw is 6/11. Continuing in this manner, the probability of drawing 7 non-yellow balls in a row is

(7/12) × (6/11) × (5/10) × (4/9) × (3/8) × (2/7) × (1/6)

Now, there are 5 yellow balls left out of a total of 5 + 7 + 3 = 15 balls. So the probability of drawing a yellow ball on the eighth draw is 5/15.

Therefore, the probability of drawing the first yellow ball on the eighth draw is

(7/12) × (6/11) × (5/10) × (4/9) × (3/8) × (2/7) × (1/6) × (5/15)

Simplifying this expression, we get

(7 × 6 × 5 × 4 × 3 × 2 × 1 × 5) / (12 × 11 × 10 × 9 × 8 × 7 × 6 × 15)

which simplifies to:

1/2772

Learn more about probability here

brainly.com/question/11234923

#SPJ4

The given question is incomplete, the complete question is:

A bag contains 3 red balls, 4 green balls, and 5 yellow balls. If balls are drawn one at a time without replacement, what is the probability that the first yellow ball is drawn on the eighth draw?

Determine the magnitude of the force P for which the resultant of the four forces acts on the rim of the plate. Given: F= 320 N. 30° 120 N 80 N P x 250 mm F 7 The magnitude of the force P is N.

Answers

The magnitude of the force P is 464.77 N.

STEP BY STEP EXPLANATION:


Step 1: Break down each force into components.
F = 320 N at 30°
Fx = F * cos(30°) = 320 * cos(30°) = 277.13 N (horizontal)
Fy = F * sin(30°) = 320 * sin(30°) = 160 N (vertical)

120 N is in the horizontal direction (assume positive x-direction):
Fx2 = 120 N

80 N is in the vertical direction (assume positive y-direction):
Fy2 = 80 N

Step 2: Sum up the components.
Total horizontal force (Fxtotal) = Fx + Fx2

= 277.13 + 120 = 397.13 N


Total vertical force (Fytotal) = Fy + Fy2

= 160 + 80 = 240 N

Step 3: Find the magnitude of the resultant force.
            Resultant force (R) = sqrt(Fxtotal^2 + Fytotal^2)

= sqrt(397.13^2 + 240^2) = 464.77 N

Step 4: Determine the magnitude of the force P.
Since the resultant of the four forces should act on the rim of the plate, it means that the force P should be equal in magnitude and opposite in direction to the resultant force R.

The magnitude of the force P is 464.77 N.

To know more about Magnitude:

https://brainly.com/question/30033702

#SPJ11

help please?!This sphere has a radius of 6 cm.

What is the surface area of the sphere?

Enter your answer, in exact form, in the box.

Answers

Answer:

Step-by-step explanation:

  It's[tex]288\pi in2[/tex]

Answer:

Step-by-step explanation:

Here is real answer :>

what's a drawback to using a histogram?

Answers

Answer:

Step-by-step explanation:

One potential drawback of using a histogram is that it can be sensitive to the choice of bin width or bin size. If the bin size is too small, the histogram may appear too noisy or have too many empty bins, which can obscure patterns in the data. If the bin size is too large, important features of the distribution may be lost or smoothed out. Additionally, histograms do not always show the actual values of the data points, but rather a summary of the data. This means that some details about the data may be lost, such as the exact values of outliers or individual data points.

the area of the figure

Answers

Answer:

432

Step-by-step explanation:

go 16x27 and that is your answer

Answer:

The area of the figureo will be 432

BECAUSE :-

You have to multiply 16 × 27 = 432

Hope Its Help You !!

SOMEONE HELP PLEASE
Kiki runs 4 3/7 miles during the first week of track practice she runs 6 2/3 miles during the second week of track practice. How much longer does kiki run during the second week of track practice than the first week of track practice

Answers

Total longer distance is [tex]3\frac{2}{21} miles[/tex], that kiki run during the second week of track practice than the first week of track practice.

We have, a runner Kiki and she runs on track for practice. In first week,

Distance runs by Kiki on track practic = [tex]4 \frac{3}{7} miles[/tex]

which is a mixed fraction so, simplify it into simple fraction that is 25/7 miles. In second week,

Distance runs by Kiki on track practice = [tex]6 \frac{2}{3} miles[/tex]

After simplification, it is equals to 20/3 miles. We have to calculate the longer distance that kiki run during the second week of track practice than the first week of track practice. Let the distance run during second week be longer by 'x miles' from distance run during first week. The required distance can be calculated by difference between the distance that kiki run during the second week of track practice and the first week of track practice. So, [tex]x = \frac{20}{3} - \frac{25}{7 }[/tex].

taking least common multiple of (3,7)= 21

=> [tex] x = \frac{20× 7 - 3× 25}{21} [/tex]

[tex] =>x = \frac{ 140 - 75}{21} = \frac{65}{21 }[/tex].

Hence, required distance value is

[tex]3\frac{2}{21} [/tex].

For more information about distance, visit :

https://brainly.com/question/28551043

#SPJ4

The base year is 2012. Real GDP in 2012 was​ $15 trillion. The GDP price index in 2019 was​ 105, and real GDP in 2019 was​ $16 trillion. What was the percentage increase in production from 2012 to​ 2019, and by what percentage did the price level rise from 2012 to​ 2019?

Answers

Real GDP increase in 2019 was $16 trillion, representing a 6.67% increase from 2012. The price level rose by 5% from 2012 to 2019, with a GDP price index of 105 in 2019.

To calculate the percentage increase in production from 2012 to 2019, we need to use the following formula:

Percentage increase in production = ((Real GDP in 2019 - Real GDP in 2012) / Real GDP in 2012) x 100

Substituting the given values, we get:

Percentage increase in production = ((16 trillion - 15 trillion) / 15 trillion) x 100

Percentage increase in production = (1 trillion / 15 trillion) x 100

Percentage increase in production = 6.67%

Therefore, there was a 6.67% increase in production from 2012 to 2019.

To calculate the percentage increase in price level from 2012 to 2019, we can use the following formula:

Percentage increase in price level = ((GDP price index in 2019 - GDP price index in 2012) / GDP price index in 2012) x 100

Substituting the given values, we get:

Percentage increase in price level = ((105 - 100) / 100) x 100

Percentage increase in price level = (5 / 100) x 100

Percentage increase in price level = 5%

Therefore, the price level rose by 5% from 2012 to 2019.

To know more about GDP increase:

https://brainly.com/question/24077817

#SPJ4

Write the ratios for sin A and cos A. The diagram is not drawn to scale.
sin A= 14/50 cos A 48/50
sin A= 48/50 cos A 14/50
sin A 48/14 cos A 14/50
sin A= 48/50 cos A 14/48

Answers

The ratio of SINA and COSA = 48/50 and 14/50

What is trignometric ratios?

This is the boundary or contour length of a 2D geometric shape.

Depending on their size, multiple shapes may have the same circumference. For example, imagine a triangle made up of wires of length L.

The same wire can be used to create a square if all sides are the same length.

The length covered by the perimeter of the shape is called the perimeter. Therefore, the units of circumference are the same as the units of length.

As we can say, the surroundings are one-dimensional. As a result, you can measure in meters, kilometers, millimeters, etc.

Inches, feet, yards, and miles are other globally recognized units of circumference measurement.

According to our question,

sina = perpendicular\ hypotenuse

= 48/50

cosa= base\ perpendicular

=

14/50

Hence, The ratio of SINA and COSA = 48/50 and 14/50

learn more about trigonometric ratios click here:

brainly.com/question/1201366

#SPJ1

What is the volume of the cone expressed in terms of pi?

Answers

Answer: V≈339.29 in

Step-by-step explanation:

ten chairs are arranged in a circle. find the number of subsets of this set of chairs that contain at least three adjacent chairs.

Answers

Ten chairs are arranged in a circle. The number of subsets of this set of chairs that contain at least three adjacent chairs is 310.

The given that 10 chairs arranged in a circle.

            Now we have to find the number of subsets of this set of chairs that contain at least three adjacent chairs.

           To solve this, we can use the concept of permutations and combinations. The first step is to consider the number of ways in which three chairs can be selected and arranged in a subset that is adjacent to each other.

           This can be done in 10 different ways, as there are 10 chairs in total and we can select any one of them as the starting point.

           The next step is to consider the number of ways in which we can add additional chairs to this subset. For example, we can add a fourth chair to the subset in two different ways: either to the left of the first chair or to the right of the third chair.

            Similarly, we can add a fifth chair to the subset in four different ways, a sixth chair in six different ways, and so on. Using this logic, we can create the following table:

                    Length of subset number of ways to select the subset number of ways to add chairs

           Total number of subsets31 (adjacent)

                                = 10  ---43 (adjacent) 10*2

                                =20---55 (adjacent)10*4

                                =40---67 (adjacent)10*6

                                =60---79 (adjacent)10*8

                               =80---810 (adjacent)10*10

                               =100---

              As we can see from the table, the total number of subsets that contain at least three adjacent chairs is given by:

           Total number of subsets = 10 + 20 + 40 + 60 + 80 + 100

                                                     = 310

       Therefore, the number of subsets of this set of chairs that contain at least three adjacent chairs is 310.

Learn more about the number of subsets of chairs at: https://brainly.com/question/17514113

#SPJ11

For any acute angle A if sin A = 2x-1/2x+1, what is the value of cos A cot A?

Answers

The trigonometric expression cosAcotA = 8x/(4x² - 1)

What is a trigonometric expression?

A trigonometric expression is an expression that contains trigonometric ratios.

Given that for any acute angle A if sin A = 2x-1/2x+1, we desire to find the value of cos A cot A?

So, we proceed as follows

cosAcotA = cosA × cosA/sinA  (since cotA = cosA/sinA)

= cos²A/sinA

Now using the trigonometric identity

sin²A + cos²A = 1

⇒ cos²A = 1 - sin²A

So, substituting this into the equation, we have that

cosAcotA = cos²A/sinA

=  (1 - sin²A)/sinA

= 1/sinA - sin²A/sinA

= 1/sinA - sinA

Substituting the value of sinA into the equation, we have

= 1/(2x - 1)/(2x + 1) - (2x - 1)/(2x + 1)

= (2x + 1)/(2x - 1) - (2x - 1)/(2x + 1)

Taking the L.C.M, (2x - 1)(2x + 1), we have

= [(2x + 1)² - (2x - 1)²]/[(2x - 1)(2x+ 1)]

=  [(2x + 1)² - (2x - 1)²]/[(2x)² - 1²)]

=  [(2x + 1 + 2x - 1)(2x + 1 - (2x - 1)]/(2x)² - 1²)

=  [(2x + 1 + 2x - 1)(2x + 1 - 2x + 1)]/4x² - 1)

=  [(4x)(2)]/4x² - 1)

= 8x/(4x² - 1)

So, cosAcotA = 8x/(4x² - 1)

Learn more about trigonometric expresion here:

https://brainly.com/question/29321762

#SPJ1

Answer

D =

E =

Please help

Answers

Answer:

d = 3.75

e = 8/3

Step-by-step explanation:

8/6 = 5/d = e/2

4/3 = 5/d

4d = 3 × 5

d = 3.75

4/3 = e/2

3e = 4 × 2

e = 8/3

Subtract the following polynomials.

Answers

The subtraction of the polynomials (3.1x + 2.8z) - (4.3x - 1.2z) is -1.2x + 4x

How to subtract polynomials?

A polynomial is an expression consisting of a sum of a finite number of terms, each term being the product of a constant coefficient and one or more variables raised to a non-negative integer power.

(3.1x + 2.8z) - (4.3x - 1.2z)

open parenthesis

3.1x + 2.8z - 4.3x + 1.2z

combine like terms

3.1x - 4.3x + 2.8z + 1.2z

-1.2x + 4x

Ultimately, -1.2x + 4x is the results of the subtraction of the polynomial.

Read more on polynomials:

https://brainly.com/question/4142886

#SPJ1

function “p” is in the form y = ax 2+c. if the values of “a” and “c” are both less than 0. which graph could represent “p”?

Answers

Since the value of a is less than 0, the graph of the parabola would be opening downwards. Because of this we can rule out option C. In a quadratic equation, c represents the y-intercept, and, in this case c is negative, meaning the y-intercept is less than 0. Only option B has a downward-opening curve and a y-intercept less than 0, so it is the answer.

Answer: p= 2+c

Step-by-step explanation:

In a basketball free-throw shooting contest shots made by Sam and Wil were in the ratio 7:9.Wil made 6 more shots than Sam. Find the number of shots made by each of them.

Answers

Sam = 7(3) = 21
Will = 9(3) = 27

Answer:

Sam made 21 shots, and Wil made 27 shots.

there are 3 soccer games in a month, and 8 are played at night. the season is 4 months. how many games are the season?

Answers

There are a total of 48 soccer games in the season.

Since there are 3 soccer games in a month, there will be 12 games in a season (3 games/month x 4 months). Since 8 games are played at night and assuming that all games are played either during the day or at night, we can calculate the number of games played during the day as:

Number of day games = Total number of games - Number of night games

= 12 games/month x 4 months - 8 night games/month x 4 months

= 48 games - 32 games

= 16 games

Therefore, the total number of games in the season is:

Total number of games = Number of day games + Number of night games

= 16 games + 32 games

= 48 games

So, there are 48 soccer games in the season.


To know more about soccer, refer here:

https://brainly.com/question/15499943#

#SPJ11

Triangle ABC has vertices at A(−4, 3), B(0, 5), and C(−2, 0). Determine the coordinates of the vertices for the image if the preimage is translated 4 units down.

A′(−4, −1), B′(0, 1), C′(−2, −4)
A′(−4, 7), B′(0, 9), C′(−2, 4)
A′(0, 3), B′(4, 4), C′(3, 0)
A′(−8, 7), B′(−4, 9), C′(−6, 4)

Answers

The coordinates of the vertices for the image if the preimage is translated 4 units down are A′(-4, -1), B′(0, 1), C′(-2, -4).

What is meant by preimage?

In geometry, a preimage is the original figure or shape before any transformation is applied. It is the initial configuration of the object that is being transformed. For example, if we have a square and we rotate it by 90 degrees, the original square is the preimage and the resulting figure after the rotation is the image.

To translate the preimage 4 units down, we need to subtract 4 from the y-coordinates of all vertices. Therefore, the coordinates of the image vertices are:

A′(-4, 3-4) = (-4, -1)

B′(0, 5-4) = (0, 1)

C′(-2, 0-4) = (-2, -4)

Therefore, the vertices of the image triangle are A′(-4, -1), B′(0, 1), and C′(-2, -4).

So, the correct option is: A′(-4, -1), B′(0, 1), C′(-2, -4).

To learn more about preimage visit:

https://brainly.com/question/30093252

#SPJ1

Please help 30 points I've been struggling

Identify the Slope and y - intercept from the graph

Slope (m) =
b =

Write the equation of the line in Slope-Intercept form

Answers

The slope is 2, and the y-intercept is 4.

a die is rolled twice and the sum of numbers appearing on the upper faces of them is observed to be 7. what is the probability that the number 2 has appeared atleast once? hint: use the concept of conditional probability)

Answers

The probability of getting at least one 2 given that the sum of the numbers is 7 is 2/6 or 1/3.

To find the probability that the number 2 has appeared at least once given that the sum of the numbers is 7, we need to use the concept of conditional probability.

Let's consider the possible outcomes when two dice are rolled. The total number of outcomes is 36, as each die has six possible outcomes.

Out of these 36 outcomes, there are six outcomes in which the sum of the numbers appearing on the upper faces is 7. These outcomes are (1,6), (2,5), (3,4), (4,3), (5,2), and (6,1).

Out of these six outcomes, there are two outcomes in which the number 2 appears at least once: (1,6) and (2,5).

We can use the formula for conditional probability to verify our answer:

P(2 appears at least once | sum is 7) = P(2 appears at least once and sum is 7) / P(sum is 7)

P(2 appears at least once and sum is 7) = 2/36 = 1/18 (as there are two outcomes with a sum of 7 that have a 2 in them)

P(sum is 7) = 6/36 = 1/6

So, P(2 appears at least once | sum is 7) = (1/18) / (1/6) = 1/3, which is consistent with our previous answer.

To learn more about probability click on,

https://brainly.com/question/13866543

#SPJ4

for the beam and loading shown, (a) draw the shear and bending-moment diagrams, (b) determine the equations of the shear and bending-moment curves. 5.1

Answers

Bending moment curve equation below point A will be:

M = 15x - 3x² for 0 ≤ x ≤ b

Determination of shear and bending moment curves.

For the beam and loading shown, we can do the following:

Equation of shear curve (above point A):V = RA - w.x

For x = a,V = RA - w.a

For x = b,V = RA - w.b

Since the loading is symmetric, RA = w(a + b) / 2= (6 * 5) / 2= 15kNV = 15 - 6a for a ≤ x ≤ b

Equation of shear curve (below point A):

V = RA - w.x

For x = 0,V = RA - w.0RA = w(a + b) / 2= (6 * 5) / 2= 15kNV = 15k for 0 ≤ x ≤ a

The shear curve equation becomes;

V = 15k for 0 ≤ x ≤ a

V = 15 - 6a for a ≤ x ≤ b

Equation of bending moment curve (above point A):

M = RAx - ½w.x²For 0 ≤ x ≤ a,

M = 15x - ½(6x²) = 15x - 3x²For a ≤ x ≤ b,

M = 15x - 6a(x - a) - ½(6x²)= 15x - 6ax + 6a² - 3x²

The bending moment curve equation above point A becomes:

M = 15x - 3x² for 0 ≤ x ≤ a

M = 15x - 6ax + 6a² - 3x² for a ≤ x ≤ b

Equation of bending moment curve (below point A):

M = RAx - ½w.x²For 0 ≤ x ≤ b,

M = 15x - ½(6x²) = 15x - 3x²

The bending moment curve equation below point A becomes;

M = 15x - 3x² for 0 ≤ x ≤ b

Learn more about Bending Moment.

brainly.com/question/30242055

#SPJ11

Other Questions
time, cost, and project data are recorded by computers on the jobsite. question 4 options: true false 2. a. There is a new family called the Grump family. Mr. Grump's rule is (x,y). Circle the rules thathis family members in red. Circle the imposters in blue.(3x,3y)(2x,y)(x,4y)(4x,4y)(6x, 5y)(9x, 9y)(0.3x, 0.3Y) most sediment on the continental shelf is derived from a. erosion on the continent b. life in the sea c. aliens d. chemical reactions in the water an example of closed-end credit is . question 2 options: incidental credit revolving check credit credit cards installment sales credit Select the true statements. Select the two correct answers. A. 1 . 01 < 0 . 99 1 . 01 < 0 . 99 B. 4 . 5 = 4 . 50 4 . 5 = 4 . 50 C. 3 . 5 < 3 . 39 3 . 5 < 3 . 39 D. 1 . 51 > 1 . 15 1 . 51 > 1 . 15 E. 2 . 09 = 2 . 9 which of the following is not a consequence of an increase in the government's budget deficit? question 3 options: exports increase while imports and all other variables are held constant private savings increases while holding everything else constant investment falls while holding everything else constant imports increase while exports and all other variables are held constant question 4 (1 point) the goal seems to be a nonstarter for the team because they have never mishandled fewer than 200 bags, let alone hitting the goal of 100 bags. based on expectancy theory, this is a(n) problem. a parallel-plate capacitor has a plate separation of 4.00 mm. 1) if the material between the plates is air, what plate area is required to provide a capacitance of 3.00 pf? (express your answer to three significant figures.) what might be a source of octane in the product mixture in this reaction? hint: you did quench the hydroboration reaction with water and let the mixture sit for a week before proceeding to the oxidation step. Corey spies a bald eagle in a tall tree. He was able to measure the angle of elevation to the bird from where he stands at 62. The leaves on the tree make it difficult for Corey to watch the bird, so he moves 10 feet further away from the tree to get a better view. Now his angle of elevation is 56.What is the height of the tree? Round your answer to the nearest foot.pls help the immense body of justified beliefs that consists of facts people learn from their own direct observations and facts they learn from others is known as their Do you agree with Unilever's decision to links it's brands with efforts to encourage healthy and environmentally sustainable behaviors? 2. Infer: On page 51, Wiesel writes about receivinghis engraved number. He states, "I became A-7713.After that I had no other name." On a non-literallevel what is he telling you about the effect of hisexperiences at the camp? You should referenceyour answer to #1 when answering this question.Book: Night a) Construct a probability distribution b) Graph the probability distribution using a histogram and describe its shape c) Find the probability that a randomly selected student is less than 20 years old. d) Find the probability that a randomly selected student's age is more than 18 years old but no more than 21 years old. LOOK AT SCREENSHOT FOR FULL QUESTION What is the key bond being formed in a Grignard reaction? A.Carbon-Magnesium B.Magnesium-BromineC.Carbon-Carbon D.Carbon-Oxygen Last year, 89 musicians attended a jazz camp, and 15 of them were bassists. What is the experimental probability that the first musician to sign up will be a bassist? How do you fine the blank for infinitely many solutions many firms choose to protect or enhance the natural environment as they go about their business activities. this practice is known as: question 94 options: green marketing natural marketing consumer marketing social marketing people who are more influenced by an external locus of control when faced with a threatening situation are more apt to: PLEASE HELPPPP asapppppppppp